0% found this document useful (0 votes)
121 views5 pages

Advanced Math Solutions Guide

This document provides solutions to 5 problems about metric spaces and compactness from a homework assignment. 1) It proves directly from the definition that the set K = {0} ∪ {1/n : n is a positive integer} is compact in R. 2) It constructs a compact set of real numbers whose limit points form a countable set. 3) It shows that Rk is separable by proving the set of rational points Qk is dense in Rk. 4) It proves that if every infinite subset of a metric space X has a limit point, then X is separable. 5) It proves that every compact metric space K has a countable base

Uploaded by

Mridul biswas
Copyright
© © All Rights Reserved
We take content rights seriously. If you suspect this is your content, claim it here.
Available Formats
Download as PDF, TXT or read online on Scribd
0% found this document useful (0 votes)
121 views5 pages

Advanced Math Solutions Guide

This document provides solutions to 5 problems about metric spaces and compactness from a homework assignment. 1) It proves directly from the definition that the set K = {0} ∪ {1/n : n is a positive integer} is compact in R. 2) It constructs a compact set of real numbers whose limit points form a countable set. 3) It shows that Rk is separable by proving the set of rational points Qk is dense in Rk. 4) It proves that if every infinite subset of a metric space X has a limit point, then X is separable. 5) It proves that every compact metric space K has a countable base

Uploaded by

Mridul biswas
Copyright
© © All Rights Reserved
We take content rights seriously. If you suspect this is your content, claim it here.
Available Formats
Download as PDF, TXT or read online on Scribd
You are on page 1/ 5

M ATH 140 A - HW 3 S OLUTIONS

Problem 1 (WR Ch 2 #12). Let K ⊂ R1 consist of 0 and the numbers 1/n, for n = 1, 2, 3, . . .. Prove
that K is compact directly from the definition (without using the Heine-Borel theorem).

Solution. Let {G α } be any open cover of K , which means each G α is an open set and together their
S
union α G α contains K . In order to prove K is compact, we must show there is a finite subcover.
Since 0 ∈ K ⊂ α G α , there is some α0 such that 0 ∈ G α0 . Now we know G α0 is an open set, and
S

an open set must contain a neighborhood of each of its points. Since 0 ∈ G α0 , there is some r > 0
such that Nr (0) ⊂ G α0 . Let N be the smallest integer that is greater than 1/r , which means N > r1 ,
1
or equivalently, r > N. Now notice that

1 1 1
r> > > > ··· ,
N N +1 N +2

or more formally,
r > d (0, N1 ) > d (0, N1+1 ) > d (0, N1+2 ) > · · · ,
1 1 1
which means that the points N , N +1 , N +2 , . . . are all in Nr (0), which is contained in G α0 . So we
have one of the open sets that contains all but a finite number of points of K . The only points
left to cover are 1, 21 , 13 , . . . , N1−1 . Since 1 is in K ⊂ α G α , there must be some α1 such that 1 ∈ G α1 .
S

Repeating this for the rest of the points left, we have sets G α2 ,G α3 , . . . ,G αN −1 containing the points
1 1 1
2 , 3 , . . . , N −1 respectively. Therefore,
N[
−1
K⊂ G αn ,
n=0

and we have found a finite subcover. Hence, K is compact.

Problem 2 (WR Ch 2 #13). Construct a compact set of real numbers whose limit points form a
countable set.

Solution. Let ½ µ ¶¯ ¾
1 1 ¯¯
E= 1 − m, n ∈ N .
2m n ¯
This is plotted below,

A more illustrative plot follows, with the x-axis representing points of E and the y-axis represent-
ing different values of m to visually separate out different groups of points.

1
1
Consider the points of the form p = 2m with m ∈ N. Any neighborhood of one of these points
1 1
of radius r > 0 will also contain the point q = 2m (1 − n ) where we choose the positive integer n
1
such that n <2 m
r , so that |p − q| = | 21m − 1 1 1
2m (1 − n )| = | 2m n | < r . Since q 6= p and q ∈ E , that means
p is a limit point, and thus E has at least a countably infinite number of limit points.
The fact that E is compact comes from its being closed (since it contains its limit points) and
bounded (since each point of E is contained in [0, 21 ]).

Problem 3 (WR Ch 2 #22). A metric space is called separable if it contains a countable dense
subset. Show that Rk is separable.

Solution. We claim that Qk is dense in Rk . To show this, let p = (p 1 , . . . , p k ) ∈ Rk . In order to show


Qk is dense, we need to show that p ∈ Qk or that p is a limit point of Qk . So if p ∈ Qk , we’re done.
If p ∉ Qk , we want to show that p is a limit point. Let Nr (p) with r > 0 be a neighborhood of p. Let
p
δ = r / n, and since Q is dense in R, we can find some q i 6= p i in Nδ (p i ) for i = 1, . . . , k. Then for
q = (q 1 , . . . , q k ) we have
s
r2 r2
q
d (p, q) = (p 1 − q 1 )2 + · · · + (p k − q k )2 < +···+ = r,
n n

so that q ∈ Nr (p), and thus p is a limit point.


Thus Qk is dense in Rk , and it is countable by Theorem 2.13, so Rk is separable.

Problem 4 (WR Ch 2 #24). Let X be a metric space in which every infinite subset has a limit point.
Prove that X is separable.

Solution. Fix δ > 0 and pick x 1 ∈ X . Having chosen x 1 , . . . , x j ∈ X , choose x j +1 ∈ X , if possible, so


that d (x i , x j +1 ) ≥ δ for i = 1, . . . , j . This is essentially covering X by disjoint d el t a-balls centered
at the points x 1 , x 2 , . . .. If we can do this forever without having any overlap, then the set {x j | j ∈ N}
is an infinite set without a limit point (since we have neighborhoods around each point that don’t

2
contain any other point in the set). But his is a contradiction, so this process must stop after a
finite number of steps, which means X can be covered by finitely many neighborhoods of radius
δ. Since we have proved this for any δ > 0, we can assign δ to be anything greater than zero and
still have the same result.
Let δ = 1. Then there are finitely many points x 1(1) , x 2(1) , . . . , x N
(1)
such that X is covered by δ-
1
neighborhoods centered at those points. Now let δ = 12 . Then there are finitely many points
x 1(2) , x 2(2) , . . . , x N
(2)
such that X is covered by δ-neighborhoods centered at those points. Repeating
2
this for any n ∈ N we let δ = 1
n and then there are finitely many points x 1(n) , x 2(n) , . . . , x N
(n)
such that
n
X is covered by δ-neighborhoods centered at those points. We now claim that the set

E = {x 1(1) , x 2(1) , . . . , x N
(1)
,
1

x 1(2) , x 2(2) , . . . , x N
(2)
,
2

x 1(3) , x 2(3) , . . . , x N
(3)
,
3

. . .}

is a countable dense subset of X . It’s countable by Theorem 2.12. It’s dense because for any point
1
x ∈ X and any neighborhood Nr (x) for r > 0, we can choose a positive integer n such that n < r,
and then either x = x i(n) for some 1 ≤ i ≤ Nn , or x is in some neighborhood N 1 (x i ) for some
n
1 ≤ i ≤ Nn because we have an open cover. That means that x i ∈ Nr (x) and x i 6= x, meaning x is a
limit point of E .

Problem 5 (WR Ch 2 #25). Prove that every compact metric space K has a countable base, and
that K is therefore separable.

1
Solution. For each n ∈ N, make an open cover of K by neighborhoods of radius n, and we have a
finite subcover by compactness, i.e.

[ N
[
K⊂ N 1 (x) =⇒ ∃ x 1 , . . . , x N ∈ K such that K ⊂ N 1 (x i )
n n
x∈K i =1

Doing this for every n ∈ N, we get a countable union of finite collections of sets, so that by Theorem
2.12, the collection of these sets, call it S, is countable.
We claim that S is a countable base for K , which is defined as a countable collection of open
sets such that for any x ∈ K and any open set G with x ∈ G, there is some V ∈ S such that x ∈ V ⊂ G.
Let x ∈ K and let G be any open set with x ∈ G. Then since G is open, there is some r > 0 such
1
that Nr (x) ⊂ G. Choose n ∈ N such that n < r /2, so that the maximal distance between points in a
1
neighborhood of radius n is r . Then there must be some i such that x ∈ N 1 (x i ) ⊂ Nr (x) because
n
any neighborhood of radius 1/n containing x cannot contain points a distance more than r away.
This shows that S is a countable base.
The second part of the question asks us to show that K is separable. Let {Vn } be our countable
base for K . For each n ∈ N, choose x n ∈ Vn , and let E = {x n |n ∈ N}. We claim that E is a countable

3
dense set, which would show that K is separable. First, note that E is clearly countable. To show
that it’s dense, we need to show that Ē = K . This is equivalent to showing that (Ē )c = ;. Now (Ē )c
is an open set because it’s the complement of a closed set, Ē . If (Ē )c is nonempty, then there is
some x ∈ (Ē )c , which is open, so since {Vn } is a base, there is some n such that x ∈ Vn ⊂ (Ē )c , which
implies that x n ∈ (Ē )c , a contradiction, because xn ∈ E ⇒ x n ∈ Ē ⇒ x n ∉ (Ē )c . Therefore,
(Ē )c = ;, so that Ē = K .

Problem 6 (WR Ch 2 #26). Let X be a metric space in which every infinite subset has a limit point.
Prove that X is compact.

Solution. By exercises 23 and 24, X has a countable base. Thus for any open cover {G α }, we can
write each G α as a union of sets from the countable base, meaning that every open cover has
a countable subcover {G n }. Assume by way of contradiction that no finite subcollection of {G n }
covers X . By setting à !c
n
[
Fn = Gi ,
i =1

we have that F n 6= ; for each n since if that were the case, X = F nc =


Sn
i =1
G i , which means {G 1 , . . . ,G n }
would be a finite subcover. Notice also that F n+1 ⊂ F n , since we are only removing points as n goes
up. Now we also have
à !c à à !!c à !c
∞ ∞ [ n ∞ [ n ∞
= X c = ;,
\ \ [ [
Fn = Gi = Gi = Gi
n=1 n=1 i =1 n=1 i =1 i =1
T
since {G n } is an open cover of X . Thus F n is empty. We now create an infinite set E by choosing
some x n ∈ F n (which is nonempty) for each n ∈ N and letting E = {x n |n ∈ N}. The only way that E
could not be infinite is if some x ∈ E were in an infinite number of sets F n , but that would make
T
F n nonempty. Since E is an infinite set, E has a limit point p (which is given in the beginning
of the problem statement). For each n, all but finitely many points of E lie in F n , so P must be
T
a limit point of F n for all n. But the F n ’s are closed, so p ∈ F n for all n, meaning that F n 6= ;, a
contradiction. Therefore, any open cover of X has a finite subcover, and thus X is compact.

Problem 7 (WR Ch 2 #29). Prove that every open set in R1 is the union of an at most countable
collection of disjoint segments.

Solution. By exercise 22, R1 is separable, and thus has a countable dense set, namely Q.
Let G ⊂ R be any open set. Then Q ∩G is a countable dense set in G by the Archimedean prop-
erty, and since G is open we can choose an open interval around every rational in G. Then G is the
union of that countable collection of intervals. However, we need to find a countable collection of
disjoint intervals. Notice that the union of any intervals which contain the same point is an inter-
val with a lower endpoint equal to the infimum of the lower endpoints of the intervals (possibly
−∞) and with an upper endpoint equal to the supremum of the upper endpoints of the intervals

4
(possibly ∞). We create a new countable collection of intervals whose union is G by the following
procedure.
Take any point in G ∩Q and take the union of all intervals in G that contain it. Call this interval
I 1 . Now take some point in (G \ I 1 ) ∩ Q and take the union of all intervals in G \ I 1 that contain it.
Repeating this process we get a countable collection of disjoint intervals I 1 , I 2 , I 3 , . . ., each of which
is in G and which together cover G.

Problem 8 (WR Ch 2 #30). If Rk = F n , where F n is a closed subset of Rk , then at least one F n


S∞
1
has a nonempty interior.
Sn
Solution. Assume by way of contradiction that each F n has an empty interior. Let Vn = i =1
Fi .
Since F 1 is closed, F 1c is open. If F 1c were empty, then F 1 = Rk , but then F 1◦ 6= ;, so instead F 1c
must be nonempty. Let K 1 be some neighborhood in F 1c such that K 1 ∩ V1 = ; (which we can
do by shrinking the neighborhood if necessary). Now if we have defined K n so that K n ∩ Vn = ;,
we define K n+1 by taking a neighborhood in K n \ F n+1 , which is nonempty because F n+1 has a
nonempty interior. By shrinking if necessary, we can ensure that K n+1 ⊂ K n . Notice again that
K n+1 ∩ Vn+1 = ;. This last property gives us that ∞
T
1 K n is disjoint from every F n . Also, since each
K n is compact and K n+1 ⊂ K n then by Theorem 2.39 we know that ∞
T
1 K n is nonempty. Thus, there
is some point ¶c
∞ µ∞
= (Rk )c = ;,
\ [
x∈ Kn ⊂ Fn
1 1

a contradiction, since the empty set cannot have a point in it.

You might also like